Last visit was: 25 Apr 2024, 20:13 It is currently 25 Apr 2024, 20:13

Close
GMAT Club Daily Prep
Thank you for using the timer - this advanced tool can estimate your performance and suggest more practice questions. We have subscribed you to Daily Prep Questions via email.

Customized
for You

we will pick new questions that match your level based on your Timer History

Track
Your Progress

every week, we’ll send you an estimated GMAT score based on your performance

Practice
Pays

we will pick new questions that match your level based on your Timer History
Not interested in getting valuable practice questions and articles delivered to your email? No problem, unsubscribe here.
Close
Request Expert Reply
Confirm Cancel
SORT BY:
Date
Tags:
Show Tags
Hide Tags
Director
Director
Joined: 23 Apr 2019
Status:PhD trained. Education research, management.
Posts: 806
Own Kudos [?]: 1807 [34]
Given Kudos: 203
Send PM
Director
Director
Joined: 03 Mar 2017
Posts: 586
Own Kudos [?]: 418 [4]
Given Kudos: 596
Location: India
Concentration: Operations, Technology
Send PM
Manager
Manager
Joined: 18 Jan 2018
Posts: 144
Own Kudos [?]: 283 [0]
Given Kudos: 107
Location: India
Concentration: Operations, International Business
GPA: 3.27
WE:Operations (Other)
Send PM
Intern
Intern
Joined: 22 Sep 2018
Posts: 42
Own Kudos [?]: 14 [0]
Given Kudos: 139
Location: Morocco
Send PM
Re: Activist: Any member of the city council ought either to vote against [#permalink]
ARIEN3228 wrote:
Question stem states: The conclusion of the activist’s argument follows logically if which one of the following is assumed?

option B is logically correct as per the passage but I don't think assumption is required. It is clear that the activist does not want voters to decide on the proposal.

Can anyone explain why D is wrong?


(D) If not every member of the city council abstains in the vote on the proposal, the matter will not be decided by the city’s voters.

I originally crossed D because it's saying that if you asked every member of the city to not abstain (vote against) then the city's voters will not have the right to decide, but the stem say that at least one member of the city council should vote against the proposal so one vote from the member of the city is sufficient to not let the the city's voters decide.

By POE, B is correct, but I agree that B is not an assumption
Intern
Intern
Joined: 07 Jan 2012
Posts: 48
Own Kudos [?]: 16 [2]
Given Kudos: 72
Location: Canada
Concentration: International Business, Entrepreneurship
GMAT Date: 04-30-2012
WE:Information Technology (Computer Software)
Send PM
Re: Activist: Any member of the city council ought either to vote against [#permalink]
2
Bookmarks
How can B be an assumption - it sounds more like a conclusion.
Senior Manager
Senior Manager
Joined: 27 Mar 2017
Posts: 274
Own Kudos [?]: 76 [0]
Given Kudos: 406
Location: Saudi Arabia
GMAT 1: 700 Q47 V39
GPA: 3.36
Send PM
Re: Activist: Any member of the city council ought either to vote against [#permalink]
B is more intent than assumption. Do we have correct OA here ? Isn't Not Every = At Least 1 ? Option D seems more assumption-like.
Director
Director
Joined: 03 Mar 2017
Posts: 586
Own Kudos [?]: 418 [1]
Given Kudos: 596
Location: India
Concentration: Operations, Technology
Send PM
Re: Activist: Any member of the city council ought either to vote against [#permalink]
1
Bookmarks
Intern
Intern
Joined: 27 Jun 2020
Posts: 13
Own Kudos [?]: 14 [0]
Given Kudos: 16
Location: India
Concentration: Finance, Leadership
Schools: ISB'22 (A)
GMAT 1: 660 Q49 V31
GMAT 2: 670 Q49 V32
GMAT 3: 680 Q49 V32
GPA: 4
WE:Corporate Finance (Computer Hardware)
Send PM
Activist: Any member of the city council ought either to vote against [#permalink]
Premise : Council members can vote in favor or against
Counter Premise : If they don't, then it goes to City people who i believe will reject
Conclusion : so the author wants at least one to vote

Author's assumption : City people will reject, should not be allowed to vote
Author does not want City people to vote as according to hmim they will reject, so assumption will follow logically if city people are not allowed to vote.

Hence B
GMAT Club Legend
GMAT Club Legend
Joined: 15 Jul 2015
Posts: 5181
Own Kudos [?]: 4653 [3]
Given Kudos: 631
Location: India
GMAT Focus 1:
715 Q83 V90 DI83
GMAT 1: 780 Q50 V51
GRE 1: Q170 V169
Send PM
Re: Activist: Any member of the city council ought either to vote against [#permalink]
3
Kudos
Expert Reply
warrior1991 wrote:
Hi warrior1991,

Let's take a (somewhat) similar situation, a quiz competition:
1. Any member of our team should answer, or not answer, the question posed to our team.
2. If every member of our team fails to answer the question posed to our team, the question passes to the next team.
4. Therefore, at least one member of our team should answer the question posed to our team. ← This is the conclusion.

The question asks us for a statement (3), an assumption that would help us plug the gap between (1) & (2) and (4).

1. Any member of our team should answer, or not answer, the question posed to our team.
2. If every member of our team fails to answer the question posed to our team, the question passes to the next team.
3. We should not allow the question posed to our team to pass to the next team.
4. Therefore, at least one member of our team should answer the question posed to our team.

Now the conclusion looks better. Why should we avoid a situation where no one on our team answers the question? To keep it from being passed to the next team. That's why at least one team member should answer the question.
Volunteer Expert
Joined: 16 May 2019
Posts: 3512
Own Kudos [?]: 6859 [4]
Given Kudos: 500
Re: Activist: Any member of the city council ought either to vote against [#permalink]
2
Kudos
2
Bookmarks
Expert Reply
warrior1991 wrote:

I agree with what AjiteshArun has written above, and I would not have thought to express the justification of (B) in such a way. As for (D), it presents, in mathematical or formal logic terms, the inverse of the given conditional statement. A bit more on that first:

Conditional Statement: If A, then B.
Inverse: If not A, then not B.
Converse: If B, then A.
Contrapositive: If not B, then not A.

These relationships may be useful to know for such questions, even if they are not strictly necessary to keep in mind for GMAT™ CR questions. In the question at hand, our conditional statement is as follows:

If all the members abstain, the matter will be decided by the city’s voters.

Choice (D) negates this, creating a could-be-true situation, rather than a must-be-true assumption that we are seeking:

If not every member of the city council abstains in the vote on the proposal, the matter will not be decided by the city’s voters.

Such a consideration goes beyond the scope of the passage. For all we know, one member of the council could vote for the proposal and another against, with the rest abstaining, and with a deadlock resulting from the council voting phase, the matter could still be decided by the voters of the city. We can only speculate. Looking at (B) again, given the constant presence of "proposal language," as I will call it—ought to, should—we can deduce that, to the activist, it is only reasonable to assume that the proposal should not be decided by the city's voters.

I hope that helps.

- Andrew
Tutor
Joined: 16 Oct 2010
Posts: 14823
Own Kudos [?]: 64924 [2]
Given Kudos: 426
Location: Pune, India
Send PM
Re: Activist: Any member of the city council ought either to vote against [#permalink]
2
Kudos
Expert Reply
Hovkial wrote:
Activist: Any member of the city council ought either to vote against the proposal or to abstain. But if all the members abstain, the matter will be decided by the city’s voters. So at least one member of the city council should vote against the proposal.

The conclusion of the activist’s argument follows logically if which one of the following is assumed?

(A) If all the members of the city council abstain in the vote on the proposal, the city’s voters will definitely decide in favor of the proposal.

(B) The proposal should not be decided by the city’s voters.

(C) No members of the city council will vote in favor of the proposal.

(D) If not every member of the city council abstains in the vote on the proposal, the matter will not be decided by the city’s voters.

(E) If one member of the city council ought to vote against the proposal, the other members should abstain in the vote on the proposal.


First note that it is an LSAT question. It asks for "conclusion of the activist’s argument follows logically if which one of the following is assumed"

We are not looking for an assumption as defined in GMAT. We are looking for a "sufficient premise", not a "necessary premise".

Activist:
- Any member of the city council ought either to vote against the proposal or to abstain.
- But if all the members abstain, the matter will be decided by the city’s voters.

Conclusion: So at least one member of the city council should vote against the proposal.

When will the conclusion follow? When we say "but the city's voters should not decide the matter". So at least one member of city council should vote against.

- Any member of the city council ought either to vote against the proposal or to abstain.
- But if all the members abstain, the matter will be decided by the city’s voters.
- The proposal should not be decided by the city’s voters.
Conclusion: So at least one member of the city council should vote against the proposal.

Makes complete sense, right? No gaps in logic.

(D) If not every member of the city council abstains in the vote on the proposal, the matter will not be decided by the city’s voters.
This means if even one member votes, the matter will not be decided by the city's voters.
Note that this is the flip side of our second premise above. If all members abstain, city voters will decide. If even one member votes, city voters will not decide. What is still missing is that "city voters should not be allowed to decide" to reach our conclusion "at least one member must vote". Hence this is not the answer.

Answer (B)
VP
VP
Joined: 14 Aug 2019
Posts: 1378
Own Kudos [?]: 846 [0]
Given Kudos: 381
Location: Hong Kong
Concentration: Strategy, Marketing
GMAT 1: 650 Q49 V29
GPA: 3.81
Send PM
Re: Activist: Any member of the city council ought either to vote against [#permalink]
What if some city council member votes in favor then conclusion will be distorted.( negate)
So is it not necessary that no council member vote against in favor. why can not this be an assumption?

Is C wrong because it doesn't mention about vote against or abstain on which the conclusion directly depends.
C is out of scope?

Please suggest. VeritasKarishma MentorTutoring AjiteshArun
Volunteer Expert
Joined: 16 May 2019
Posts: 3512
Own Kudos [?]: 6859 [2]
Given Kudos: 500
Re: Activist: Any member of the city council ought either to vote against [#permalink]
2
Kudos
Expert Reply
itsSKR wrote:
What if some city council member votes in favor then conclusion will be distorted.( negate)
So is it not necessary that no council member vote against in favor. why can not this be an assumption?

Is C wrong because it doesn't mention about vote against or abstain on which the conclusion directly depends.
C is out of scope?

Please suggest. VeritasKarishma MentorTutoring AjiteshArun

Yes, choice (C) is out of scope. As VeritasKarishma mentioned above, the question stem is asking us to put a finger on an assumption that allows the argument to "follow logically." There seems to be a disconnect if we attempt to justify (C) as a necessary assumption that leads to the argument. Try it out. In the vein of what AjiteshArun has done above, where would you place the sentence to create such a logical progression of ideas, given that the first and last lines are fixed? (The first line is fixed because it introduces the idea of voting on the proposal; the last line is fixed because it is the argument itself.)

Version 1:
1) Activist: Any member of the city council ought either to vote against the proposal or to abstain.
2) No members of the city council will vote in favor of the proposal.
3) But if all the members abstain, the matter will be decided by the city’s voters.
4) So at least one member of the city council should vote against the proposal.

The activist is giving a recommendation, not assessing how council members will actually vote. The second sentence seems out of place.

Version 2:
1) Activist: Any member of the city council ought either to vote against the proposal or to abstain.
2) But if all the members abstain, the matter will be decided by the city’s voters.
3) No members of the city council will vote in favor of the proposal.
4) So at least one member of the city council should vote against the proposal.

Again, the proposed assumption breaks up the linear logic that leads to the argument. Notice that what is now sentence 2 operates within a conditional (if-[then]) statement, so it is presenting a hypothetical scenario. Sentence 3, on the other hand, is definitive, not to mention that reading 3) and 4) back to back leaves me scratching my head. (How does the argument necessarily follow from the previous sentence? The ideas seem to be going in different directions, and 3) only seems to be interrupting.)

I hope that helps. If not, you seem to have three Experts willing to chime in on this one.

- Andrew
Director
Director
Joined: 03 Mar 2017
Posts: 586
Own Kudos [?]: 418 [0]
Given Kudos: 596
Location: India
Concentration: Operations, Technology
Send PM
Re: Activist: Any member of the city council ought either to vote against [#permalink]
AjiteshArun wrote:
warrior1991 wrote:
Hi warrior1991,

Let's take a (somewhat) similar situation, a quiz competition:
1. Any member of our team should answer, or not answer, the question posed to our team.
2. If every member of our team fails to answer the question posed to our team, the question passes to the next team.
4. Therefore, at least one member of our team should answer the question posed to our team. ← This is the conclusion.

The question asks us for a statement (3), an assumption that would help us plug the gap between (1) & (2) and (4).

1. Any member of our team should answer, or not answer, the question posed to our team.
2. If every member of our team fails to answer the question posed to our team, the question passes to the next team.
3. We should not allow the question posed to our team to pass to the next team.
4. Therefore, at least one member of our team should answer the question posed to our team.

Now the conclusion looks better. Why should we avoid a situation where no one on our team answers the question? To keep it from being passed to the next team. That's why at least one team member should answer the question.



Thanks for the explanation.

So do you mean to say that option B is bridging the gap firmly than is option D??
GMAT Club Legend
GMAT Club Legend
Joined: 15 Jul 2015
Posts: 5181
Own Kudos [?]: 4653 [1]
Given Kudos: 631
Location: India
GMAT Focus 1:
715 Q83 V90 DI83
GMAT 1: 780 Q50 V51
GRE 1: Q170 V169
Send PM
Activist: Any member of the city council ought either to vote against [#permalink]
1
Kudos
Expert Reply
warrior1991 wrote:
Thanks for the explanation.

So do you mean to say that option B is bridging the gap firmly than is option D??
Hi warrior1991,

Let's take another look at the question stem:
The conclusion of the activist’s argument follows logically if which one of the following is assumed?

This question is asking us to provide an assumption that would plug the logical gap between the support and the conclusion. The basic situation is a little bit like "if we do X, Y won't happen. So we should do X". But why? Why should we stop Y from happening?

In the quiz example, this is the argument that D would lead to:
1. Any member of our team should answer, or not answer, the question posed to our team.
2. If every member of our team fails to answer the question posed to our team, the question passes to the next team.
3. If not every member of team abstains, the question will not pass to the next team.
4. Therefore, at least one member of our team should answer the question posed to our team.

But this leaves us no closer to plugging that gap we identified. Why exactly should at least one person answer the question? Why is that so important? For example, what if there are negative points for answering incorrectly, and answering a question incorrectly is worse than letting the next team take a shot at the question? The situation in the question is the same. We can do something to stop voters from deciding the issue. But that alone does not tell us why we should do that something. The correct option tells us that voters should not be allowed to decide on the proposal, and that gives us a reason to take the action advocated by the conclusion.

I hope the quiz example helped, but I'll drop it so that we can focus on the original question, more directly addressed by this post and this post.
Director
Director
Joined: 21 Feb 2017
Posts: 521
Own Kudos [?]: 1039 [0]
Given Kudos: 1091
Location: India
GMAT 1: 700 Q47 V39
Send PM
Activist: Any member of the city council ought either to vote against [#permalink]
(A) If all the members of the city council abstain in the vote on the proposal, the city’s voters will definitely decide in favor of the proposal.
-When you see extreme language, you should get suspicious, because a choice with extreme language is almost never correct on the GMAT.

(B) The proposal should not be decided by the city’s voters.
- logically correct fills the gap in the argument.

(D) If not every member of the city council abstains in the vote on the proposal, the matter will not be decided by the city’s voters.
- here, we need to ask, is this really helping us bridge the gap in the argument? keep in mind assumptions are unstated premises. we NEED the assumptions to be true for the argument to stand.
Current Student
Joined: 25 Apr 2020
Posts: 59
Own Kudos [?]: 76 [1]
Given Kudos: 138
Location: India
GMAT 1: 790 Q50 V51
GRE 1: Q167 V163
GPA: 3
Send PM
Activist: Any member of the city council ought either to vote against [#permalink]
1
Kudos
IMO the city people get to decide when there is no result achieved by the council. This can happen in two ways.
1. No one in the council votes.(given in passage)
2. Equal votes in favor and against.(not given in passage but follows logically)
Thus D is out. B is correct.

[size=80][b][i]Posted from my mobile device[/i][/b][/size]
Intern
Intern
Joined: 16 Aug 2021
Posts: 32
Own Kudos [?]: 14 [0]
Given Kudos: 1
Location: Australia
GMAT 1: 710 Q49 V38
GPA: 2
Send PM
Re: Activist: Any member of the city council ought either to vote against [#permalink]
Hi all,

Just out of curiosity, is B both a sufficient AND necessary assumption? If we negate B, then there won't be any reason for the conclusion to hold true. Thank you.
Manager
Manager
Joined: 14 Sep 2019
Posts: 52
Own Kudos [?]: 15 [0]
Given Kudos: 45
Location: United States
GMAT 1: 750 Q49 V42
Send PM
Re: Activist: Any member of the city council ought either to vote against [#permalink]
An assumption bridges the gap between premise and conclusion. Here there is a sentence missing between the premise and the conclusion that the at least one vote should be against the proposal so that the city voters don't vote. That sentence is option B- The proposal should not be decided by the city’s voters, a statement that bridges the gap and hence is the assumption.

Note that the question is specifically asking what is necessary for the conclusion to follow, if you negate option B- The proposal can be decided by the city voters, the conclusion doesn't follow.
A-There is no way to know what the city's members will decide
C- Out of Scope
D- Wrong, only one member needs to vote against
E- Out of Scope
GMAT Club Bot
Re: Activist: Any member of the city council ought either to vote against [#permalink]
Moderators:
GMAT Club Verbal Expert
6921 posts
GMAT Club Verbal Expert
238 posts
CR Forum Moderator
832 posts

Powered by phpBB © phpBB Group | Emoji artwork provided by EmojiOne